N-oscillator system with two sides attached

In summary, Homework Equations attempted to add imaginary oscillators to the system in order to create one oscillator every a/2 length. However, they were unable to solve the normal mode frequency and amplitude.
  • #1
Miles123K
57
2

Homework Statement


ced3de86-4b72-4ab8-89e3-957b47072760

The system is shown in the image. In the beaded string shown in Figure 1, the interval between neighboring beads is a, and the distance from the end beads to the wall is a/2. All the beads have mass m and are constrained to move only vertically in the plane of the paper. The strings are massless with constant string tension T.

Homework Equations


I am sure that I'm supposed to construct an analogous infinite system first. That system would have standard translation symmetry matrix and should be solved in that way. Boundary conditions need to be applied later to make the system limited.

The Attempt at a Solution


I made a wild attempt to try to add imaginary oscillators to the system so that there will be one oscillator every a/2 length. I am pretty sure I am very wrong since an N-oscillator system should not have 2N normal modes.
 

Attachments

  • upload_2019-2-2_20-23-17.png
    upload_2019-2-2_20-23-17.png
    29.4 KB · Views: 406
Physics news on Phys.org
  • #2
I gave some extra thoughts and managed to get sub-question A and B, and have a potential solution (don't know right or wrong) for the normal mode frequency, but I couldn't solve the the normal modes amplitude.
 
  • #3
It would help if you wrote out some of your expressions explicitly so that we know what notation you are using, etc. When you have the usual case where the "zeroth" particle is held fixed, the normal mode displacement, ##u_n##, of the beads can be written as ##u_n = \cos (\omega t) \sin (np)## where n is an integer labeling the beads and ##p## is to be determined from the boundary conditions. The reason for using the sine function here is to satisfy the boundary condition for the case where the ##n = 0## bead is fixed.

In your case, you want the ##n = 0## bead to "mirror" the ##n = 1## bead: ##u_0 = -u_1## at all times. Try writing ##u_n = \cos (\omega t) \sin (np+\phi)## where ##\phi## is a phase consant. Can you choose ##\phi## so that the boundary condition ##u_0 = -u_1## is satisfied at all times?
 
  • #4
TSny said:
It would help if you wrote out some of your expressions explicitly so that we know what notation you are using, etc. When you have the usual case where the "zeroth" particle is held fixed, the normal mode displacement, ##u_n##, of the beads can be written as ##u_n = \cos (\omega t) \sin (np)## where n is an integer labeling the beads and ##p## is to be determined from the boundary conditions. The reason for using the sine function here is to satisfy the boundary condition for the case where the ##n = 0## bead is fixed.

In your case, you want the ##n = 0## bead to "mirror" the ##n = 1## bead: ##u_0 = -u_1## at all times. Try writing ##u_n = \cos (\omega t) \sin (np+\phi)## where ##\phi## is a phase consant. Can you choose ##\phi## so that the boundary condition ##u_0 = -u_1## is satisfied at all times?
The problem is for some reason I couldn't choose any ##\phi## that also satisfies the conditions of the other wall (##u_N = -u_(N+1)##). I did get ##\omega## though, If I am correct that should be ##\omega_n = 2 \omega_0 \sin((n\pi)/(N+1))## where n varies from 1 to N+1?
 
  • #5
Miles123K said:
The problem is for some reason I couldn't choose any ##\phi## that also satisfies the conditions of the other wall (##u_N = -u_(N+1)##).
Use the boundary condition at the left end to find ##\phi##. Then use the boundary condition at the right end to determine the allowed values of ##p## in ##u_n = \cos (\omega t) \sin (np+\phi)##

.I did get ##\omega## though, If I am correct that should be ##\omega_n = 2 \omega_0 \sin((n\pi)/(N+1))## where n varies from 1 to N+1?
Did you mean to write that n varies from 1 to N, instead of 1 to N +1? There should only be N independent modes.

I don't quite get the same expression for ##\omega_n##. I believe the fact that the phase constant ##\phi## is nonzero in your problem will modify the expression for ##\omega_n## compared to the case where the n = 0 and n = N +1 beads are fixed in place.
 
  • #6
TSny said:
Use the boundary condition at the left end to find ##\phi##. Then use the boundary condition at the right end to determine the allowed values of ##p## in ##u_n = \cos (\omega t) \sin (np+\phi)##

Did you mean to write that n varies from 1 to N, instead of 1 to N +1? There should only be N independent modes.

I don't quite get the same expression for ##\omega_n##. I believe the fact that the phase constant ##\phi## is nonzero in your problem will modify the expression for ##\omega_n## compared to the case where the n = 0 and n = N +1 beads are fixed in place.
Yes I mean from 1 to N, my bad. I feel the same way actually. That's why I remain in doubt of my answers...
 
  • #7
So far, you haven't shown how you are getting your answers. If you show your work, it is more likely that we can help.
 
  • #8
I did another attempt using the methods from this lecture:
Provided boundary conditions ##y_0 = - y_1##, ##y_N = - y_{N+1}##,
Incorporate the system as if it's a part of an infinite system, and the infinite system is subject to translation symmetry matrix ##S##, and ##S \vec A = \beta \vec A ##
I wrote the motion equation ## m \ddot y_p = \frac T a y_{p-1} - \frac 2T a y_p + \frac T a y_{p+1}## Note that ##T## is tension and ##a## is distance between the beads
We know that any system subject to translation symmetry matrix ##S## has normal mode amplitude ## \vec A = \begin{bmatrix}
\beta \\
\beta^2 \\
\beta^3 \\
\vdots
\end{bmatrix} ## whereas ## \beta = e^{ika}##
define ##\omega_0^2 = \frac T {ma}## ##m## as mass
##M^{-1}K \vec A = \beta \vec A## so, for individual terms of ##\vec A##
##\omega^2 A_{j} = \omega_0^2 (-A_{j-1}+2A_j-A_{j+1})##
##\omega^2 \beta^j = \omega_0^2 (-\beta^{j-1}+2\beta^j-\beta^{j+1})##
so ##\omega^2 = \omega_0^2 (-\beta^{-1}+2-\beta^{1})##
Recall that ## \beta = e^{ika}##
##\omega^2 = 2 \omega_0^2 (1-\cos(ka))##
because of the nature of this equation, ##k## and ##-k## have identical solutions of ##\omega##
Following the step of the professor, amplitude should be a linear combination of ##\beta## for ##k## and ##-k##
##\beta^j = A_j = c_1e^{ijka} + c_2e^{-ijka}##
To apply the boundary conditions here, I would have for ##A_0 = A_1##, ##A_N = - A_{N+1}##,
So, ##c_1+c_2 = -c_1 e^{ika} - c_2e^{-ika}## (eq.1)
and ##c_1e^{iNka}+c_2e^{-iNka} = -c_1 e^{i(N+1)ka} - c_2e^{-i(N+1)ka}## (eq.2)
My attempt at solving this weird system of equations is to use the fact that since both equations can be rearranged be equal to zero and equation 2 can be reduced, I made the equations like this with all the ##c_2## terms canceled out. Next, I just rearranged to make the equation zero.
##c_1 +c_1 ce^{ika} = c_1 e^{i(2N)ka} + c_1e^{i(2N+1)ka} = c_1e^{i2Nka}(1+e^{ika}) ##
Keep rearranging and I got:
##(1-e^{i2Nka})c_1(1+e^{ika})=0##
To solve for k that can take different values, ##(1-e^{i2Nka})=0##
so ##2Nka = n2\pi## and ##k = \frac {n\pi} {Na}##, substituting ##k## back to ##\omega^2 = 2 \omega_0^2 (1-\cos(ka))## would give me the normal modes frequencies but I don't know how to solve for the amplitudes...

I'm actually in High School so I am really doing this all by my own. I sent emails to MIT asking for solutions of their pset and they said this course material is too recent that they are still using them so solutions aren't uploaded. Please don't judge if everything is screwed up.
 
  • #9
TSny said:
So far, you haven't shown how you are getting your answers. If you show your work, it is more likely that we can help.
Ah Yes. I just uploaded my solution.
 
  • #10
OK, Miles. You are to be commended for tackling this material while still in high school! :oldsmile:

So, you are working with complex exponentials. Your equations (1) and (2) below look good:
Miles123K said:
##c_1+c_2 = -c_1 e^{ika} - c_2e^{-ika}## (eq.1)
##c_1e^{iNka}+c_2e^{-iNka} = -c_1 e^{i(N+1)ka} - c_2e^{-i(N+1)ka}## (eq.2)
I also believe you now have the correct result for the allowed values of ##k##:
##k = \frac {n\pi} {Na}##
The denominator has a factor of ##N##, rather than ##N+1##. Good. So, I think this will give you the correct values for ##\omega##.

To get the amplitudes of the beads for a particular mode, use your (eq.1) for the boundary condition at the left end. See if you can show that this yields ##c_2 = -c_1 e^{ika}##.

Then use this result in ##\beta^j = c_1e^{ijka} + c_2e^{-ijka}##. Can you manipulate it into the following?

##\beta^j = c_1 e^{ika/2} \left[e^{i(j-1/2)ka} - e^{-i(j-1/2)ka} \right] = c_1e^{ika/2} 2i \sin\left[(j-1/2)ka \right]##

Since ##c_1## is an arbitrary complex constant, you can just lump together all the stuff in front of the sine function as a new constant, ##C##.
 
  • Like
Likes Miles123K
  • #11
TSny said:
OK, Miles. You are to be commended for tackling this material while still in high school! :oldsmile:

So, you are working with complex exponentials. Your equations (1) and (2) below look good:

I also believe you now have the correct result for the allowed values of ##k##:
The denominator has a factor of ##N##, rather than ##N+1##. Good. So, I think this will give you the correct values for ##\omega##.

To get the amplitudes of the beads for a particular mode, use your (eq.1) for the boundary condition at the left end. See if you can show that this yields ##c_2 = -c_1 e^{ika}##.

Then use this result in ##\beta^j = c_1e^{ijka} + c_2e^{-ijka}##. Can you manipulate it into the following?

##\beta^j = c_1 e^{ika/2} \left[e^{i(j-1/2)ka} - e^{-i(j-1/2)ka} \right] = c_1e^{ika/2} 2i \sin\left[(j-1/2)ka \right]##

Since ##c_1## is an arbitrary complex constant, you can just lump together all the stuff in front of the sine function as a new constant, ##C##.

Thanks so much! That perfectly resolves my problem.
 

1. What is an N-oscillator system with two sides attached?

An N-oscillator system with two sides attached refers to a physical or mathematical model consisting of N oscillators that are connected on two sides. This means that each oscillator is connected to two other oscillators, creating a network or chain-like structure.

2. What is the purpose of studying N-oscillator systems with two sides attached?

The study of N-oscillator systems with two sides attached allows scientists to better understand complex systems and behaviors that arise from the interactions between individual components. This type of system is commonly used to model various phenomena such as synchronization, pattern formation, and chaos.

3. How does the behavior of an N-oscillator system with two sides attached differ from that of a single oscillator?

The behavior of an N-oscillator system with two sides attached is more complex and can exhibit emergent properties that are not present in a single oscillator. This is because the interactions between the oscillators can lead to self-organization and the emergence of collective behaviors.

4. What are some real-world applications of N-oscillator systems with two sides attached?

N-oscillator systems with two sides attached have been used to model a wide range of phenomena in various fields such as physics, biology, and chemistry. Some examples include the synchronization of fireflies, the formation of traffic patterns, and the behavior of neurons in the brain.

5. How do scientists study N-oscillator systems with two sides attached?

Scientists use a combination of mathematical models, computer simulations, and experimental techniques to study N-oscillator systems with two sides attached. These methods allow them to analyze the behavior of the system and make predictions about its dynamics and emergent properties.

Similar threads

  • Advanced Physics Homework Help
Replies
1
Views
2K
  • Advanced Physics Homework Help
Replies
2
Views
3K
  • Introductory Physics Homework Help
Replies
5
Views
1K
  • Advanced Physics Homework Help
Replies
2
Views
1K
  • Advanced Physics Homework Help
Replies
3
Views
5K
  • Advanced Physics Homework Help
Replies
1
Views
2K
  • Introductory Physics Homework Help
Replies
12
Views
3K
  • Advanced Physics Homework Help
Replies
22
Views
2K
Replies
1
Views
781
  • Advanced Physics Homework Help
Replies
3
Views
4K
Back
Top